Momento angular de representación matricial

Se supone que debemos dar una representación matricial de L S para un electrón con yo = 1 y s = 1 2 .

Yo leo L S como L S . ¿Es esto correcto? Entonces tendríamos por ejemplo para

L S ( | 1 , 1 | 1 / 2 , 1 / 2 ) = L | 1 , 1 S | 1 / 2 , 1 / 2 = 2 | 1 , 1 3 4 | 1 / 2 , 1 / 2 = 3 2 2 | 1 , 1 | 1 / 2 , 1 / 2 .

¿Es correcta esta corrección? En ese caso, ¿debería proceder de esta manera con todos los demás vectores base y escribir los valores propios en la diagonal de una matriz?

L y S son vectores. L S es un escalar. Qué es L S ? Si desea utilizar el producto tensorial de operadores, la expresión correcta es L S = k = 1 3 L i S i .
¿Entonces dices que esto es una tontería? bueno, ¿sabes cómo obtener una representación matricial para L S ?
Sí: reemplazar cada operador L i y S i para la matriz correspondiente. Usa el hecho de que yo = 1 y s = 1 / 2 para encontrar estas matrices en los libros de texto.
no entiendo esto si decimos que j = L + S , ¿no significa esto que j = L I d + I d S ? Creo que no he entendido todo el concepto de suma de momentos angulares. ¿Cuál es la notación matemática correcta para j = L + S ?
La notación matemática correcta para esa suma es j = L 1 girar + 1 orbe S , donde el 1 s son operadores de identidad en los espacios de spin y orbitales de Hilbert.
j = L + S en realidad significa: j k = L k I s pag i norte + I o r b S k para k = 1 , 2 , 3 .
Gracias. Entonces, ¿solo puedo definir este producto tensorial para cada componente? Pero, ¿cómo me ayuda esto a evaluar este L S Quiero decir, solo sé lo que L 3 S 3 hace sobre la base canónica en Mecánica Cuántica, por supuesto L 1 S 1 y lo mismo con el índice 2 están completamente indeterminados por el principio de incertidumbre?
Por lo tanto L S = k ( L k I ) ( I S k ) = k L k S k .
Solo debe escribir explícitamente la forma matricial de L S , por lo que yo entendí. Esta matriz existe sin importar si no hay vectores propios comunes para todos los componentes del espín o momento angular.
sí, pero ¿cómo encuentro esta matriz? ¿Qué necesito hacer ahora?
Con respecto a las matrices de espín, como s = 1 / 2 , ellos son / 2 σ i ¿DE ACUERDO?
Sobre L i , ya que sabes que yo = 1 , son exactamente las mismas matrices que determinan los componentes de espín de una partícula de espín 1 . Están escritos en casi todos los libros de texto.
en.wikipedia.org/wiki/Pauli_matrices , sección Mecánica cuántica las tres primeras matrices (hay yo = j y L i = j i .

Respuestas (1)

Hay dos problemas a tratar que deben ser desenredados para resolver problemas como estos.

  • Ambos operadores de momento angular son operadores vectoriales , por lo que en cierto sentido "toman valores" en R 3 ; se le pide su producto punto, que debe tomarse dentro de esa copia de R 3 . Tendrías el mismo problema si te pidieran calcular el producto escalar r pag para una sola partícula sin espín.

  • Los operadores de momento angular orbital y de espín actúan sobre los dos factores diferentes de un producto tensorial de espacios de Hilbet. Por lo tanto, cualquier producto (operador) de un operador orbital escalar con un operador de espín escalar debe interpretarse como un producto tensorial. Tendrías el mismo problema si te pidieran calcular el producto L 2 S 2 , lo que debe interpretarse como L 2 S 2 .

Así, en su caso, debe leer L S como

L S = i = 1 3 L i S i = i = 1 3 L i S i .
Para calcular la representación matricial de esto, debe comenzar con la representación matricial de cada L i y S i . Luego calcula las matrices de productos tensoriales L i S i . Finalmente, suma todas esas matrices para obtener el resultado final.

Todo esto queda mucho más claro con un ejemplo. El z componente, por ejemplo, es fácil, ya que cada matriz está dada por

L z = ( 1 0 0 0 0 0 0 0 1 ) y S z = 2 ( 1 0 0 1 ) ,
en las bases { | 1 , | 0 , | 1 } y { | 1 2 , | 1 2 } respectivamente. La matriz del producto tensorial, entonces, en la base { | 1 | 1 2 , | 0 | 1 2 , | 1 | 1 2 , | 1 | 1 2 , | 0 | 1 2 , | 1 | 1 2 } , es dado por
L z S z = 2 2 ( 1 ( 1 0 0 0 0 0 0 0 1 ) 0 ( 1 0 0 0 0 0 0 0 1 ) 0 ( 1 0 0 0 0 0 0 0 1 ) 1 ( 1 0 0 0 0 0 0 0 1 ) ) = 2 2 ( 1 0 0 0 0 0 0 0 0 0 0 0 0 0 1 0 0 0 0 0 0 1 0 0 0 0 0 0 0 0 0 0 0 0 0 1 ) .
Este procedimiento debe repetirse tanto con el X y el y componentes Cada uno de ellos producirá una matriz de seis por seis (en este caso). Para obtener su respuesta final, debe sumar las tres matrices.